[obm-l] Re: [obm-l] Re: [obm-l] Dúvida e ajuda.

2022-04-08 Por tôpico Pedro José
Grato a todos! Já, já tenho de voltar ao trabalho. Depois dou uma olhada. Mas achei a demonstração usando casa de pombos, simples e prática. Já que tem de haver um p/q com pp temos w=x+p/q, onde x é a parte inteira de w/q, então pq e os restos só podem q-1, uma hora tem de repetir e aí volta a sequ

[obm-l] Re: [obm-l] Re: [obm-l] Dúvida

2021-11-22 Por tôpico Pedro José
Boa tarde! Grato, pela ajuda! Não conheço. Vou abrir um leque de estudo para tentar entender! Valeu a curiosidade, com o que cheguei consegui matar o problema. Genericamente, consegui que a solução levaria a uma expressão que era um quadrado perfeito,esse era o objetivo. Só que me deu curiosidade,

[obm-l] Re: [obm-l] Re: [obm-l] Re: [obm-l] Dúvida

2019-12-05 Por tôpico Bernardo Freitas Paulo da Costa
Oi Pedro e Pedro, e demais colegas da OBM-L Eu também nunca lera a definição de elipses através da razão entre as distâncias. Achei interessante, porque talvez permita "interpolar" entre elipses, parábolas e hipérboles. Mas até hoje, todas as definições que eu vira de elipses (inclusive a da som

[obm-l] Re: [obm-l] Re: [obm-l] Dúvida

2019-12-04 Por tôpico Pedro José
Boa noite! As retas são cônicas degeneradas. Mas são cônicas. Definição de cônica : Dada duas retas g,l concorrentes (cuja interseção é {V} no |R3 que não sejam perpendiculares e um plano Pi. A interseção desse plano com o cone K, reto de vértice V e eixo l , obtido pela rotação da reta g ao redo

[obm-l] Re: [obm-l] Re: [obm-l] Re: [obm-l] Dúvida basica equação polar

2019-09-02 Por tôpico Ralph Teixeira
Pois bem, se voce parametrizar com relacao ao centro, teria x(teta)=1+cos(teta) e y(teta)=sin(teta). Se fosse assim, teria que ser 0 wrote: > Caro Ralf, obrigado pela resposta.Para mim ficou confuso pq pensei que a > parametrização do círculo se daria colocando como referencia o novo centro > do m

[obm-l] Re: [obm-l] Re: [obm-l] Dúvida basica equação polar

2019-09-02 Por tôpico Gabriel Lopes
Caro Ralf, obrigado pela resposta.Para mim ficou confuso pq pensei que a parametrização do círculo se daria colocando como referencia o novo centro do mesmo. Quando penso em circulos diferentes , por exemplo residindo em apenas um quadrante tenho dificuldade de imaginar varrendo todos os pontos . V

[obm-l] Re: [obm-l] Re: [obm-l] Re: [obm-l] Dúvida

2019-05-24 Por tôpico Anderson Torres
Em dom, 19 de mai de 2019 às 13:24, Pedro José escreveu: > Bom dia! > Anderson, > obrigado. Porém faltou-me saber se os entendimentos anteriores estão > corretos. > O texto não tinha nenhum glossário para ajudar, ou uma referência do gênero? Alguns bons livros de Teoria dos Números, em especial

[obm-l] Re: [obm-l] Re: [obm-l] Dúvida

2019-05-19 Por tôpico Pedro José
Bom dia! Anderson, obrigado. Porém faltou-me saber se os entendimentos anteriores estão corretos. Grato, PJMS Em sáb, 18 de mai de 2019 13:27, Anderson Torres < torres.anderson...@gmail.com escreveu: > > > Em sex, 17 de mai de 2019 às 10:49, Pedro José > escreveu: > >> Bom dia! >> >> Tenho uma

[obm-l] Re: [obm-l] Re: [obm-l] dúvida sobre a OBMU

2019-01-20 Por tôpico Pedro Soares
Combinatória aproveita bastante. E pra exemplificar o que pode ter em comum, esse ano o problema 6 do Nível U também estava na prova do nível 3 (não sei o número do problema) On Sat, 19 Jan 2019 at 09:42, Anderson Torres wrote: > Em sáb, 12 de jan de 2019 às 16:41, Luiz Kv > escreveu: > > > > O

[obm-l] Re: [obm-l] Re: [obm-l] Dúvida conceitual (equações)

2018-10-15 Por tôpico Pedro José
Boa tarde! Artur, não sou contrário a multiplicidade da raiz. Porém, mesmo coma a multiplicidade, a raiz continua sendo única. Todavia,não há como negar, facilita sobremaneira as relações de Girard, para soma e produto é fácil de ajeitar, mas quando passamos a somatório de produtos dois a dois, trê

[obm-l] Re: [obm-l] Re: [obm-l] Re: [obm-l] Re: [obm-l] Dúvida conceitual (equações)

2018-10-15 Por tôpico Bernardo Freitas Paulo da Costa
On Mon, Oct 15, 2018 at 8:07 AM Claudio Buffara wrote: > > Derivando e igualando a zero o lado esquerdo da sua equação, ficamos com: > -2*cos(x)*sen(x) + sen(x) = 0 ==> > sen(x) = 0 ou cos(x) = 1/2 ==> > x = 0 ou x = pi ou x = 2pi > ou x = pi/3 ou x = 5pi/3. > > Assim, uma definição que me parec

Re: [obm-l] Re: [obm-l] Re: [obm-l] Dúvida conceitual (equações)

2018-10-15 Por tôpico Kevin Felipe Kühl Oliveira
Exatamente nisso que estava pensando. Se fizessemos 4^x = y teriamos uma equação polinomial de grau 3, ai fica mais evidente a existência de múltiplas raizes. Abraços Kevin Kühl On 15 Oct 2018 07:25 -0300, Claudio Buffara , wrote: > Qual a soma das raizes de (2^x - 8)^3 = 0? > Se a equação acim

Re: [obm-l] Re: [obm-l] Re: [obm-l] Re: [obm-l] Dúvida conceitual (equações)

2018-10-15 Por tôpico Claudio Buffara
Derivando e igualando a zero o lado esquerdo da sua equação, ficamos com: -2*cos(x)*sen(x) + sen(x) = 0 ==> sen(x) = 0 ou cos(x) = 1/2 ==> x = 0 ou x = pi ou x = 2pi ou x = pi/3 ou x = 5pi/3. Assim, uma definição que me parece adequado para equações em geral (e não necessariamente polinomiais)

Re: [obm-l] Re: [obm-l] Re: [obm-l] Re: [obm-l] Dúvida conceitual (equações)

2018-10-15 Por tôpico Claudio Buffara
Pensando só como uma equação, talvez faça sentido não considerar a multiplicidade. Mas, no seu exemplo, no intervalo [0,2pi], os gráficos de f(x) = cos(x) - 1/2 e de g(x) = (cos(x) - 1/2)^2 tem um comportamento bem distinto um do outro em vizinhanças de pi/3 e 5pi/3. Por exemplo, o gráfico de f

[obm-l] Re: [obm-l] Re: [obm-l] Re: [obm-l] Dúvida conceitual (equações)

2018-10-15 Por tôpico Vanderlei Nemitz
Claudio: Eu ficaria com a mesma dúvida! Pensaria em apenas uma raiz. Qual é a soma das raízes da equação (cos x)^2 - cos x + 1/4 = 0 no intervalo [0, 2pi]? Em seg, 15 de out de 2018 07:00, Claudio Buffara escreveu: > Qual a soma das raizes de (2^x - 8)^3 = 0? > Se a equação acima fosse apresent

Re: [obm-l] Re: [obm-l] Re: [obm-l] Dúvida conceitual (equações)

2018-10-15 Por tôpico Claudio Buffara
Qual a soma das raizes de (2^x - 8)^3 = 0? Se a equação acima fosse apresentada como: 2^(3x) - 24*2^(2x) + 192*2^x - 512 = 0, isso mudaria sua resposta? Enviado do meu iPhone Em 15 de out de 2018, à(s) 00:29, Vanderlei Nemitz escreveu: > Valeu, Pedro! Tomara que mais alguém emita sua opinião

[obm-l] Re: [obm-l] Re: [obm-l] Dúvida conceitual (equações)

2018-10-14 Por tôpico Vanderlei Nemitz
Valeu, Pedro! Tomara que mais alguém emita sua opinião. Um abraço! Em dom, 14 de out de 2018 18:59, Pedro José escreveu: > Boa noite! > Bom questionamento. Vou me posicionar na arquibancada. > Minha posição é controversa. Se quer se levar em conta a repetição tem que > se falar do produto das ra

[obm-l] Re: [obm-l] Re: [obm-l] Dúvida

2018-08-23 Por tôpico Pedro José
Boa noite! É fato. Grato, PJMS. Em Qua, 22 de ago de 2018 23:00, Ralph Teixeira escreveu: > Acho que nao... Ah, se eu entendi corretamente, (3,6,9) e (3,5,12) seria > um contra-exemplo. > > Abraco, Ralph. > > > On Wed, Aug 22, 2018 at 8:06 PM Pedro José wrote: > >> Boa noite. >> >> Sejam duas s

[obm-l] Re: [obm-l] Re: [obm-l] Dúvida

2018-05-31 Por tôpico Pedro José
Bom dia! Corrigindo uma grande bobagem, confirme me alertado. A ordem de 10 nos 11 é 2 e não 1. Mas como 2|6, não muda nada. Saudações, PJMS Em Sex, 25 de mai de 2018 14:37, Pedro José escreveu: > Boa tarde! > Creio ter conseguido. > Criei um número com fatores congruentes a 1 mod 6, exceto o 5

[obm-l] Re: [obm-l] Re: [obm-l] Dúvida

2018-05-25 Por tôpico Pedro José
Boa tarde! Creio ter conseguido. Criei um número com fatores congruentes a 1 mod 6, exceto o 5 e o11. Além disso a ordem de 10 mod desses fatores é sempre 6, exceto o 5 e o 11 que será 1, melhor. Mas o 5 não tem problema. Então o objetivo é firmar um número da seguinte forma: ...AB...B.

[obm-l] Re: [obm-l] Re: [obm-l] Dúvida

2018-05-24 Por tôpico Pedro José
Boa noite! Minha primeira tentativa foi tudo 1. Mas aí a soma dos quadrados também é 1001=7*11*13. As ordens de 10 mod desses fatores são 6, 1 e 6. Mas têm 1001 algarismos e aí 6 ł 1001não serve. Tentei outros arranjos com grupos de algarismos iguais, mas sem sucesso. Mas o que não compreendo é por

[obm-l] Re: [obm-l] Re: [obm-l] Re: [obm-l] Re: [obm-l] Dúvida num Enunciado

2018-04-26 Por tôpico Pedro José
Boa tarde! Bernardo, Realmente eu falhei. Fiquei com a expressão |x+3| < 4 na cabeça. Até uso um delta, e comento que não pode ser maior que 4. Saudações, PJMS Em 25 de abr de 2018 22:33, "Jaare Oregim" escreveu: > > > 2018-04-25 21:30 GMT-03:00 Bernardo Freitas Paulo da Costa < > bernardo...@gm

[obm-l] Re: [obm-l] Re: [obm-l] Re: [obm-l] Dúvida num Enunciado

2018-04-25 Por tôpico Jaare Oregim
2018-04-25 21:30 GMT-03:00 Bernardo Freitas Paulo da Costa < bernardo...@gmail.com>: > 2018-04-25 20:41 GMT-03:00 Claudio Buffara : > > O [...] > "Determine r > 0 tal que [ |x+3| < r => (A^2 - 10A + 9 > 0 para todo A > real) ]." > > Que continua com o "problema" de ter um "x" livre. Daí, a propos

[obm-l] Re: [obm-l] Re: [obm-l] Re: [obm-l] Re: [obm-l] Dúvida num Enunciado

2018-04-25 Por tôpico Claudio Buffara
Verdade! Reparei agora que deve ser r > 0. Então provavelmente o "para todo x real" não deveria estar lá. Neste caso, vira um problema com mais cara de EM: Achar todos os r > 0 tais que SE x pertence ao intervalo (-3-r , -3+r ) ENTÃO x^2 - 10x + 9 > 0 x^2 - 10x + 9 > 0 sss x pertence a (-inf,

[obm-l] Re: [obm-l] Re: [obm-l] Re: [obm-l] Re: [obm-l] Dúvida num Enunciado

2018-04-25 Por tôpico Luiz Antonio Rodrigues
Olá, Bernardo! Boa noite! Vou tentar fazer a resolução graficamente... Muito obrigado! Um abraço! Luiz On Wed, Apr 25, 2018, 9:55 PM Pedro José wrote: > Boa noite! > Cláudio, > o problema tem restrição r>0. Não dá para seguir nessa linha de r< 0. > Saudações, > PJMS > > Em 25 de abr de 2018 21:4

[obm-l] Re: [obm-l] Re: [obm-l] Re: [obm-l] Dúvida num Enunciado

2018-04-25 Por tôpico Pedro José
Boa noite! Cláudio, o problema tem restrição r>0. Não dá para seguir nessa linha de r< 0. Saudações, PJMS Em 25 de abr de 2018 21:42, "Bernardo Freitas Paulo da Costa" < bernardo...@gmail.com> escreveu: > 2018-04-25 20:20 GMT-03:00 Pedro José : > > Boa tarde! > > Realmente o enunciado está mal fe

[obm-l] Re: [obm-l] Re: [obm-l] Dúvida num Enunciado

2018-04-25 Por tôpico Bernardo Freitas Paulo da Costa
2018-04-25 20:20 GMT-03:00 Pedro José : > Boa tarde! > Realmente o enunciado está mal feito. > > Se |x+3| < r, não pode ser para todo o Real. Na verdade é x pertence a |R. > > x^2 -10x + 9 >0 ==> x pertence a A = (-oo, 1) U (9,oo) > > então temos que escolher r de modo que quando resolvamos |x + 3

[obm-l] Re: [obm-l] Re: [obm-l] Dúvida num Enunciado

2018-04-25 Por tôpico Bernardo Freitas Paulo da Costa
2018-04-25 20:41 GMT-03:00 Claudio Buffara : > O consequente (x^2 - 10x + 9 > 0 para todo x real) é falso (tome qualquer x > no intervalo [1,9]). > > Logo, para a implicação ser verdadeira, o antecedente ( |x+3| < r ) deve ser > falso, o que ocorre se e somente se r < 0. > > É mais ou menos a mesm

[obm-l] Re: [obm-l] Re: [obm-l] Dúvida num Enunciado

2018-04-25 Por tôpico Luiz Antonio Rodrigues
Olá, Claudio! Boa noite! Eu não havia percebido que o consequente é falso... Preciso ficar mais atento! Muito obrigado pela ajuda! Um abraço! Luiz On Wed, Apr 25, 2018, 8:49 PM Claudio Buffara wrote: > O consequente (x^2 - 10x + 9 > 0 para todo x real) é falso (tome qualquer > x no intervalo [1

[obm-l] Re: [obm-l] Re: [obm-l] Dúvida num Enunciado

2018-04-25 Por tôpico Luiz Antonio Rodrigues
Olá, Pedro! Boa noite! O resultado é esse mesmo. Agora eu entendi o que o problema pede. Muito obrigado! Um abraço! Luiz On Wed, Apr 25, 2018, 8:29 PM Pedro José wrote: > Boa tarde! > Realmente o enunciado está mal feito. > > Se |x+3| < r, não pode ser para todo o Real. Na verdade é x pertence a

[obm-l] Re: [obm-l] Re: [obm-l] Re: [obm-l] Re: [obm-l] Dúvida em uma solução (conjunto denso)

2017-07-10 Por tôpico Pedro Soares
Sim, é uma prova por absurdo. ''...o autor parte de uma hipótese contrária ao resultado pra chegar num absurdo...'' 2017-07-11 1:03 GMT-03:00 Bernardo Freitas Paulo da Costa < bernardo...@gmail.com>: > 2017-07-10 18:56 GMT+03:00 Antonio Carlos : > > Entendi. Muito obrigado, Pedro! > > Tem um pro

[obm-l] Re: [obm-l] Re: [obm-l] Re: [obm-l] Dúvida em uma solução (conjunto denso)

2017-07-10 Por tôpico Bernardo Freitas Paulo da Costa
2017-07-10 18:56 GMT+03:00 Antonio Carlos : > Entendi. Muito obrigado, Pedro! Tem um problema muito sério, que os logs são diferentes... log_2 3 = log(3)/log(2) = 1.5849625007211563 log_3 6 = log(6)/log(3) = 1.6309297535714573 Mas o problema está, provavelmente, na primeira hipótese (que ela tam

[obm-l] Re: [obm-l] Re: [obm-l] Dúvida em uma solução (conjunto denso)

2017-07-10 Por tôpico Antonio Carlos
Entendi. Muito obrigado, Pedro! On Jul 10, 2017 06:26, "Pedro Soares" wrote: > u/v < log_2 3 => u/v < log_3 6 , logo ou log_2 3 é menor ou igual a log_3 > 6 ou o intervalo [log_3 6, log_2 3] não possui nenhum número racional. > > u/v < log_3 6 => u/v < log_2 3 , logo ou log_3 6 é menor ou igual

[obm-l] Re: [obm-l] Re: [obm-l] Re: [obm-l] Re: [obm-l] Dúvida sobre a Obm U

2016-07-26 Por tôpico Carlos Gomes
Um bom livro é Razvan Gelca, Titu Andreescu-Putnam and Beyond (2007) Cgomes. Em 26 de julho de 2016 08:57, Otávio Araújo escreveu: > Não, onde posso conseguir? e do que ela trata? > > Em 25 de julho de 2016 11:32, Carlos Victor > escreveu: > >> >> >> >> Oi Otávio, >> >> Você já viu a Revista M

[obm-l] Re: [obm-l] Re: [obm-l] Re: [obm-l] Dúvida sobre a Obm U

2016-07-26 Por tôpico Otávio Araújo
Não, onde posso conseguir? e do que ela trata? Em 25 de julho de 2016 11:32, Carlos Victor escreveu: > > > > Oi Otávio, > > Você já viu a Revista Matemática Universitária da SBM ? > > Em 25/07/2016 10:09, Otávio Araújo escreveu: > > > > Pois é, se algum professor com experiência em olimpíadas, c

[obm-l] Re: [obm-l] Re: [obm-l] Dúvida sobre a Obm U

2016-07-25 Por tôpico Carlos Victor
Oi Otávio, Você já viu a Revista Matemática Universitária da SBM ? Em 25/07/2016 10:09, Otávio Araújo escreveu: > Pois é, se algum professor com experiência em olimpíadas, como o Nicolau por > exemplo, respondesse minha pergunta seria de grande ajuda > > Em 24 de jul de 2016, às 23:25,

[obm-l] Re: [obm-l] Re: [obm-l] Re: [obm-l] Re: [obm-l] Dúvida sobre a Obm U

2016-07-25 Por tôpico Tiago Sandino
Égua ma, sou mais ou menos da UFC, de qualquer forma, começar matemática UFC prox ano. Fiz olimpíada um tempo, imergi totalmente nisso. Fiz e trabalhei com engenharia elétrica uns anos, larguei o curso no final pq o negócio na engenharia era próprio e precisava de tempo. Atualmente tô dando aula de

Re: [obm-l] Re: [obm-l] Re: [obm-l] Re: [obm-l] Dúvida sobre a Obm U

2016-07-25 Por tôpico Otávio Araújo
Égua Tiago, eu também sou do Ceará mas meu celular atualmente não tem chip Mas tu é da UFC Tiago? E ainda estou esperando algum professor com experiência em olimpíadas de matemática responder a minha pergunta > Em 25 de jul de 2016, às 13:38, Tiago Sandino > escreveu: > > Oi pessoal. >

[obm-l] Re: [obm-l] Re: [obm-l] Re: [obm-l] Dúvida sobre a Obm U

2016-07-25 Por tôpico Tiago Sandino
Oi pessoal. Tem diversos livros de olimpíadas para graduandos (undergrads) ou com capítulos de temas exclusivamente (até onde eu saiba) universitários. Grátis na net, que eu saiba, tem muita coisa no AOPS. Dois links aqui: 1) *Fórum*: https://www.artofproblemsolving.com/community/c7_college_math 2)

[obm-l] Re: [obm-l] Re: [obm-l] Dúvida sobre a Obm U

2016-07-25 Por tôpico Raul Alves
Também tenho interesse na OBMU, e a 1ª fase tá chegando. Se algum professor puder organizar algum material de apoio, seria de grande ajuda Em 25 de julho de 2016 10:09, Otávio Araújo escreveu: > > > Pois é, se algum professor com experiência em olimpíadas, como o Nicolau > por exemplo, respondes

[obm-l] Re: [obm-l] Re: [obm-l] Dúvida em Geometria Espacial

2016-06-08 Por tôpico Daniel Rocha
Muito Obrigado, Grande Carlos !!! Em 8 de junho de 2016 20:13, Carlos Gomes escreveu: > Vc pode fazer assim: > > área total = 60 ==> 2.pi.r^2+2.pi.r.h=60 ==>h=(60-2.pi.r^2)/(2.pi.r) (*) > Por outro lado o volume é > > V=pi.r^2.h > > substituindo a expressão (*) do h , segue que > > V=60r-2.pi

[obm-l] Re: [obm-l] Re: [obm-l] Dúvida em Logaritmos

2016-06-07 Por tôpico Daniel Rocha
Muito Obrigado (mais uma vez), Carlos !!! Em 6 de junho de 2016 22:02, Carlos Gomes escreveu: > log[(sqrt 2)^(x-2)] = x ==> > (x-2)log(sqrt 2) = x ==> > x=2log(sqrt2)/(log(sqrt2)-1). > > Cgomes. > > Em 6 de junho de 2016 19:23, Daniel Rocha > escreveu: > >> Alguém poderia, por favor, solucionar

[obm-l] Re: [obm-l] Re: [obm-l] Re: [obm-l] Dúvida em Geometria Plana

2016-06-05 Por tôpico Carlos Gomes
De nada amigo! Sempre um prazer qdo posso ajudar! Abraço, Cgomes. Em 2 de junho de 2016 19:03, Daniel Rocha escreveu: > Muito Obrigado, Carlos !!! > > Em 2 de junho de 2016 18:54, Carlos Gomes escreveu: > >> Seja x a medida do ângulo BAC. Como o triângulo APQ é isosceles de base >> AP, segue q

[obm-l] Re: [obm-l] Re: [obm-l] Dúvida em Geometria Plana

2016-06-02 Por tôpico Daniel Rocha
Muito Obrigado, Carlos !!! Em 2 de junho de 2016 18:54, Carlos Gomes escreveu: > Seja x a medida do ângulo BAC. Como o triângulo APQ é isosceles de base > AP, segue q a medida do ângulo APQ também é x. Note que o ângulo BQP é > externo ao triângulo APQ, portanto, mede x+x=2x. Agora como o triâng

[obm-l] Re: [obm-l] Re: [obm-l] Dúvida "aparentemente" simples

2015-08-11 Por tôpico Pedro José
Boa tarde! Pela definição, simétrico ou oposto de um elemento a de um anel é o elemento do anel que operado com a por + resulte 0. Portanto o simétrico ou oposto de zero é zero. Saudações, PJMS. Em 11 de agosto de 2015 12:02, Ralph Teixeira escreveu: > Acho que a convencao de quase todos eh qu

[obm-l] Re: [obm-l] Re: [obm-l] Re: [obm-l] dúvida

2015-07-09 Por tôpico Israel Meireles Chrisostomo
Obrigado Ralph Em 9 de julho de 2015 12:37, Ralph Teixeira escreveu: > Vamos generalizar para R^n: com a noção usual (Euclideana) de comprimento, > o comprimento do segmento que liga (x1,x2,...,xn) a (y1,y2,...,yn) é: > > d=raiz((y1-x1)^2+(y2-x2)^2+...+(yn-xn)^2) > > Esta é a noção usual de dist

[obm-l] Re: [obm-l] Re: [obm-l] dúvida

2015-07-09 Por tôpico Ralph Teixeira
Vamos generalizar para R^n: com a noção usual (Euclideana) de comprimento, o comprimento do segmento que liga (x1,x2,...,xn) a (y1,y2,...,yn) é: d=raiz((y1-x1)^2+(y2-x2)^2+...+(yn-xn)^2) Esta é a noção usual de distância entre dois pontos -- confira que é o que você conhece na reta (n=1) e no pla

[obm-l] Re: [obm-l] Re: [obm-l] Dúvida Análise(tannery's theorem)

2015-05-05 Por tôpico Israel Meireles Chrisostomo
Obrigado a ambos, as suas respostas são ambas interessantes.Em particular quero agradecer ao Ralph, que mesmo depois de eu o contrapor em argumentos(que por sinal eram infundados) em uma outa pergunta, mesmo assim respondeu com paciência minha dúvida Em 5 de maio de 2015 10:40, Ralph Teixeira esc

[obm-l] RE: [obm-l] Re: [obm-l] Dúvida(questão simples)

2014-02-22 Por tôpico marcone augusto araújo borges
Obrigado! Date: Sat, 22 Feb 2014 00:31:24 -0300 Subject: [obm-l] Re: [obm-l] Dúvida(questão simples) From: tarsise...@gmail.com To: obm-l@mat.puc-rio.br Erramos juntos. Pq tb achei 58. -- Esta mensagem foi verificada pelo sistema de antivírus e acredita-se estar livre de perigo.

[obm-l] RE: [obm-l] Re: [obm-l] Dúvida(questão simples)

2014-02-22 Por tôpico marcone augusto araújo borges
Eu pensei assim também.Obrigado! From: ilhadepaqu...@bol.com.br To: obm-l@mat.puc-rio.br Subject: [obm-l] Re: [obm-l] Dúvida(questão simples) Date: Sat, 22 Feb 2014 00:28:27 -0300 quando ele anda no sentido horário ele anda 380 graus em 40 minutos porque o ponteiro das horas em 40 minutos

[obm-l] Re: [obm-l] RE: [obm-l] RE: [obm-l] Dúvida Indução

2012-05-17 Por tôpico Ralph Teixeira
Cuidado: ao passar de n=k para n=k+1 no Passo de Inducao... o ultimo termo "era" 3n-1, agora eh 3(n+1)-1=3n+2 -- nao eh questao de "somar um no termo", eh "trocar n por n+1". Abraco, Ralph 2012/5/17 Thiago Bersch : > Então eu estava tentando fazer mas parava no mesmo ponto, fazia > 2+5+8

[obm-l] RE: [obm-l] RE: [obm-l] Dúvida Indução

2012-05-16 Por tôpico Thiago Bersch
Então eu estava tentando fazer mas parava no mesmo ponto, fazia 2+5+8+...+(3n-1)+[(3n-1)+1], chegando aí eu me perco From: joao_maldona...@hotmail.com To: obm-l@mat.puc-rio.br Subject: [obm-l] RE: [obm-l] Dúvida Indução Date: Mon, 14 May 2012 15:24:47 -0300 Vamos dizer que para n respeite a

[obm-l] Re: [obm-l] Re: [obm-l] Dúvida de geometria analítica: É perpendicular, portanto... (?)

2012-03-22 Por tôpico Felipe Blassioli
Muito obrigado. Fiquei perdido porque o livre fez essa passagem do "portanto" sem fazer referência explícita à operação vetorial e eu não tinha conhecimento algum de álgebra linear, por isso fiquei perdido, mas agora deu pra situar ^^ 2012/3/22 Eduardo Wilner > Antes de mais nada seria interess

[obm-l] RE: [obm-l] Re: [obm-l] Dúvida

2012-03-12 Por tôpico Vanessa Nunes de Souza
Obrigada!Vanessa Nunes Date: Mon, 12 Mar 2012 17:04:46 -0300 Subject: [obm-l] Re: [obm-l] Dúvida From: tarsise...@gmail.com To: obm-l@mat.puc-rio.br a) Fazendo-se a diferença entre (100c+10b+a) - (100a+10b+c) = 396 99c-99a=396 c-a=4 Sendo 2c=a => a=4 e c=8, como a,b,c formando uma PA, temos q

[obm-l] Re: [obm-l] Re: [obm-l] Dúvida de Lógica

2012-02-08 Por tôpico Ralph Teixeira
H nao sei nao. Vou usar "C" para "estah contido" e "E" para "pertence a". Concordo que toda inclusao de conjuntos pode ser pensada como uma implicacao (bom, com um quantificador "para todo"). Afinal: A C B eh o mesmo que dizer para todo x, xEA => xEB Por isso, concordo que a Teoria dos C

[obm-l] Re: [obm-l] RE: [obm-l] Dúvida em gabarito

2011-11-27 Por tôpico JOSE AIRTON CARNEIRO
Sua resposta está correta. A área da figura em questão é formada por um triângulo equilátero + 2 segmentos circulares de arcos 60°. = a^2(4pi - 3raiz(3))/12. Airton Em 26 de novembro de 2011 15:19, João Maldonado escreveu: > > Deixa eu ver se entendi > > > Dado um quadrado, construa dois circu

[obm-l] Re: [obm-l] Re: [obm-l] Re: [obm-l] Re: [obm-l] Dúvida

2011-11-02 Por tôpico Kleber Bastos
) = 1, é válido: f(a/b) =... -- *De:* Kleber Bastos *Para:* obm-l@mat.puc-rio.br *Enviadas:* Quarta-feira, 2 de Novembro de 2011 22:21 *Assunto:* [obm-l] Re: [obm-l] Re: [obm-l] Dúvida É isso mesmo: Mostrar que ∀ nº racional a/b>0, M.D.C.(a,b)=1 é válida a sentença: f(a/b)=f(1)^a/b (...

[obm-l] Re: [obm-l] Re: [obm-l] Re: [obm-l] Dúvida

2011-11-02 Por tôpico Joao Maldonado
Novembro de 2011 22:21 Assunto: [obm-l] Re: [obm-l] Re: [obm-l] Dúvida É isso mesmo: Mostrar que ∀ nº racional a/b>0, M.D.C.(a,b)=1 é válida a sentença: f(a/b)=f(1)^a/b ( f(1) elevado a a/b) Em 2 de novembro de 2011 20:57, Victor Hugo Rodrigues escreveu: Como assim? Acho que falta algo aí

[obm-l] Re: [obm-l] Re: [obm-l] Dúvida

2011-11-02 Por tôpico Kleber Bastos
É isso mesmo: Mostrar que ∀ nº racional a/b>0, M.D.C.(a,b)=1 é válida a sentença: f(a/b)=f(1)^a/b ( f(1) elevado a a/b) Em 2 de novembro de 2011 20:57, Victor Hugo Rodrigues < victorhcr.victorh...@gmail.com> escreveu: > Como assim? Acho que falta algo aí. > > Em 2 de novembro de 2011 17:17, Klebe

[obm-l] Re: [obm-l] Re: [obm-l] Dúvida

2011-10-24 Por tôpico Julio César Saldaña
O mesmo visto de outro modo: Lucas e Pedro tem a mesma velocidade. Então quando lucas sai da ponte, pedro percorriou 2/5 da ponta, ou seja falta ainda 1/5 da ponte. Então o trem percorre a ponte inteira no mesmo tempo que pedro percorre 1/5 da ponte, ouseja o trem é 5 vezes mais veloz que pedro

[obm-l] RE: [obm-l] Re: [obm-l] dúvida - teoria dos números

2011-08-05 Por tôpico Rhilbert Rivera
Pequeno Teorema de Fermat: a^(p-1) ==1(mod p), se mdc(a,p)=1. Como 47 é primo e mdc(2,47) =1, então 2^46 ==1 (mod 47). É claro que podemos dizer ( de acordo com as propriedades das potências nas congruências) que 2^23==1 (mod 47), o que nos leva 2^23 -1 ==0(mod 47). Date: Sun, 31 Jul 2011 16

[obm-l] Re: [obm-l] Re: [obm-l] RE: [obm-l] Re: [obm-l] Re: [obm-l] dúvida sobre séries

2011-06-08 Por tôpico Artur Costa Steiner
O critério mais simples para mostrar que a série harmônica diverge talvez seja o baseado no seguinte teorema: Se x_n é uma sequência decrescente de reais tal que Soma x_n converge, então lim n x_ n = 0. (Prove isto) Se x_n = 1/n, x_n decresce para 0 mas lim n x_n = 1, o que mostra que Soma x_n di

[obm-l] Re: [obm-l] RE: [obm-l] Re: [obm-l] Re: [obm-l] dúvida sobre séries

2011-06-07 Por tôpico Rodrigo Renji
Olá! Então acho bem bacana esse também ( e nem é tão complicado de demonstrar, eu acho ) Esse critério pode ser usado para estudar a convergência de [ SOMA de 1/ k^p ] também pois [ SOMA de 2^k / 2^(kp) ] = [ SOMA de 2^(k (1-p)) ] se 1 - p< 0, isto é 1< p a série converge por série geom

[obm-l] RE: [obm-l] Re: [obm-l] Re: [obm-l] dúvida sobre séries

2011-06-07 Por tôpico Luís Lopes
Sauda,c~oes, Legal este critério, parece ter sido criado para a série harm. E a esse respeito, o autor da pergunta poderia ler também sobre a constante de Euler. []'s Luís > Date: Mon, 6 Jun 2011 23:50:37 -0300 > Subject: [obm-l] Re: [obm-l] Re: [obm-l] dúvida sobre sé

[obm-l] Re: [obm-l] Re: [obm-l] dúvida sobre séries

2011-06-06 Por tôpico Rodrigo Renji
Olá! Uma outra maneira ( além da que os colegas enviaram antes), para mostrar que a série não converge, tem um critério de convergência que acho legal, Critério de condensação de Cauchy: Se x_k é uma sequência decrescente de termos positivos ( como é o caso de 1/k ) então a série [ SOMA de

[obm-l] Re: [obm-l] Re: [obm-l] Re: [obm-l] Dúvida - OBM Nível Universitário

2011-01-24 Por tôpico Tiago
Seria uma olimpiada mais dificil ainda de ganhar, haha. 2011/1/24 Hugo Fernando Marques Fernandes > Oi, Bruna. > > Pois é, eu já tinha ouvido dizer isso e queria confirmar. > É uma pena, mas fazer o que? Regras são regras... > > Talvez fosse o caso de criar uma categoria nova pra quem já tem dip

[obm-l] Re: [obm-l] Re: [obm-l] Dúvida - OBM Nível Universitário

2011-01-24 Por tôpico Hugo Fernando Marques Fernandes
Oi, Bruna. Pois é, eu já tinha ouvido dizer isso e queria confirmar. É uma pena, mas fazer o que? Regras são regras... Talvez fosse o caso de criar uma categoria nova pra quem já tem diploma, né? Obrigado pela resposta. Hugo. Em 24 de janeiro de 2011 02:20, Bruna Campos escreveu: > PS.: E só

[obm-l] Re: [obm-l] Re:[obm-l] Dúvida-Geometria ana lítica

2010-10-11 Por tôpico Ralph Teixeira
Eu pensei dum jeito mais "desenho geométrico". No fundo, é a mesma solução do Marcone acima, só pensada de outro jeito: Faça uma figura com A, B e a reta y=6 onde mora C. Novamente, o ponto-chave da questão é D, o médio de BC. Como C está na reta y=6 e B=(4,0), o médio D está na reta y=3 (homotet

[obm-l] RE: [obm-l] Re: [obm -l] Dúvida-Geometria ana lítica

2010-10-11 Por tôpico marcone augusto araújo borges
Eu tambem estranhei essas medidas mas penso q o triângulo existe.A mediana de BC divide o triângulo ABC em dois de mesma àrea((12/2)=6 unidades).Seja D o pé da referida mediana.A área do triângulo ABD=(1/2)*4*3*senÂ=6.Dai,senÂ=1 e  é um ângulo reto,então BD=CD=5 implica BC=10.Nesse caso,a proj

[obm-l] Re: [obm-l] RE: [obm-l] Re: [obm-l] dúvida sobre le ma de teoria dos números.

2010-06-05 Por tôpico Bernardo Freitas Paulo da Costa
2010/6/5 Lucas Hagemaister > >  Hum... Entendi. Obrigado! > O que mais ou menos o lema quer dizer é o seguinte: > Sempre que termos m|a e n|a, onde mn|a, m e n serão primos entre si. Tivermos, para não assassinar o português. E não, cuidado com a ordem das implicações. A e B => C não quer dizer q

[obm-l] RE: [obm-l] Re: [obm -l] dúvida sobre lem a de teoria dos núme ros.

2010-06-05 Por tôpico Lucas Hagemaister
Hum... Entendi. Obrigado! O que mais ou menos o lema quer dizer é o seguinte: Sempre que termos m|a e n|a, onde mn|a, m e n serão primos entre si. O que eu fiz foi o contrário(ali no caso do 4 e 10): Sempre que termos m e n primos entre si, onde m|a e n|a, mn|a. Como vimos, no caso do 4 e

[obm-l] Re: [obm-l] Re: [obm-l] Dúvida sobre estimadores te ndenciosos e não-tendenciosos

2009-11-16 Por tôpico Marcelo Salhab Brogliato
Olá, Ralph, chegamos aos mesmos valores e conclusões. Entendi perfeitamente o problema dos "100% de amostras 0", hehehe. Muito obrigado pela resposta, grande abraço, Salhab 2009/11/16 Ralph Teixeira > Oi, Marcelo. > > Não sou expert nisso não, mas eu faria do mesmo jeito que você fez... > Em

[obm-l] Re: [obm-l] Re: [obm-l] Dúvida sobre limites

2009-10-28 Por tôpico Paulo Barclay Ribeiro
 Luiz. Creio que o erro que cometi foi ter dividido o calculo desse limite em duas partes( produto dos limites 1+n/n+1 com sen(npi/2), nesta parte fiz: -1 escreveu: De: Luiz Paulo Assunto: [obm-l] Re: [obm-l] Dúvida sobre limites Para: obm-l@mat.puc-rio.br Data: Quarta-feira, 28 de Outubro de

[obm-l] Re: [obm-l] RE: [obm-l] Re: [obm-l] dúvida de inter pretação

2009-05-14 Por tôpico Marcelo Costa
*HUMILDEMENTE PEÇO DESCULPAS AOS MEMBROS DA LISTA POR FUGIR AOS PROPÓSITOS DA MESMA, E AGRADEÇO A BOA VONTADE DO PROF. PALMERIM. DORAVANTE TOMAREI MAIS CUIDADO AO APRESENTAR PROBLEMAS PARA QUE SEJAM PERTINENTES AOS PROPÓSITOS DA LISTA. * 2009/5/12 Albert Bouskela > Olá Palmerim, > > > > Obrigad

[obm-l] RE: [obm-l] Re: [obm-l] dúvida d e interpretação

2009-05-12 Por tôpico Albert Bouskela
Olá Palmerim, Obrigado pela citação! Sua resposta está correta e didática. Não obstante, vou pedir-lhe um favor: acho que deveríamos parar de elucidar dúvidas tais como a que foi apresentada pelo Marcelo. Acredito que seja prudente preservar o propósito desta Lista: a discussão de proble

[obm-l] Re: [obm-l] Re: [obm-l] Dúvida em questão de R aciocínio CORRIGINDO

2009-02-03 Por tôpico João Luís
hahahahahhahahahahhaha tá certo seu puxão de orelha Nehab. Mas convenhamos, não seria bem melhor se cada questão que fosse postada aqui viesse com os dados de sua origem? Abração a todos, João Luís. - Original Message - From: Carlos Nehab Cc: obm-l@mat.puc-rio.br Sent: Tuesd

[obm-l] Re: [obm-l] Re: [obm-l] Dúvida em questão de R aciocínio

2009-02-03 Por tôpico João Luís
Obrigado Nehab - Original Message - From: Carlos Nehab To: obm-l@mat.puc-rio.br Sent: Tuesday, February 03, 2009 11:27 AM Subject: Re: [obm-l] Re: [obm-l] Dúvida em questão de Raciocínio Oi, João, Foi numa prova da ANPAD. Em geral boas questões... http://www.anpadcur

[obm-l] Re: [obm-l] Re: [obm-l] Dúvida em questão de Racio cínio

2009-02-03 Por tôpico Ralph Teixeira
Fiz como o João Luís falou, deu certo: faça um diagrama de Venn, preencha "de dentro para fora". Chamando os conjuntos de A, B e C, (chamo "e" de interseção, "ou" de união): A e B e C = (A e B) e (A e C) = {Cão} (usando II e IV) A e B e (não C) = (A e B) - (A e B e C) = {Boi} (II) A e (não B) e C

[obm-l] RES: [obm-l] Re: [obm-l] Re: [obm-l] Dúvida com questão

2008-09-16 Por tôpico Bouskela
AIL PROTECTED] _ De: [EMAIL PROTECTED] [mailto:[EMAIL PROTECTED] Em nome de João Luís Enviada em: terça-feira, 16 de setembro de 2008 13:01 Para: obm-l@mat.puc-rio.br Assunto: [obm-l] Re: [obm-l] Re: [obm-l] Dúvida com questão Sim, é verdade. Ficou incompleto mesmo. O q

[obm-l] Re: [obm-l] Re: [obm-l] Dúvida com questão

2008-09-16 Por tôpico João Luís
Sim, é verdade. Ficou incompleto mesmo. O que acontece é que eu quis enfatizar que, independentemente da incompletude do enunciado, a bicondicional dada será falsa. E, com isso, acabei me esquecendo do sinal do termo quadrático. Obrigado pela observação, Bouskela. - Original Message -

[obm-l] Re: [obm-l] RE: [obm-l] Dúvida em trigonomet ria e nos Complexos

2008-07-22 Por tôpico Paulo Mello
Victor, valew! Vou aplicar as sua dicas e resolver os problemas. Muito obrigado pela sua atenção.   Um grande abraço.   Paulo Mello. = --- Em ter, 22/7/08, [EMAIL PROTECTED] <[EMAIL PROTECTED]> escreveu: De: [EMAIL PROTECTED] <[EMAIL

Re: [obm-l] Re: [obm-l] Re:[obm-l] dúvida (Q uadriláteros)

2006-05-05 Por tôpico cleber vieira
Valeu claudio, a idéia de fazer DO = OX  e daí provar que X coincide com H foi um xeque-mate no problema, parabéns e muito obrigado pela sua resolução. Abraços Cleber  Abra sua conta no Yahoo! Mail - 1GB de espaço, alertas de e-mail no celular e anti-spam realmente eficaz.

[obm-l] Re: [obm-l] RE: [obm-l] dúvida fatorial

2006-04-12 Por tôpico Ronaldo Luiz Alonso
Um conjunto A de funcoes analiticas, duas a duas distintas, definidas em C ( C e o conjunto dos numeros complexos ) tal que para cada z pertencente a C fixado, o conjunto { f(z), f variando em A} seja enumeravel. Pergunto : A e um conjunto enumeravel ? E respondo propondo um exercicio : P

[obm-l] Re: [obm-l] Re: [obm-l] dúvida fatorial

2006-04-04 Por tôpico Ronaldo Luiz Alonso
Qualquer valor diferente de "um" atribuído por "convenção" estaria negando a definição de fatorial. SE considerarmos a interpretação de fatorial como número de bijeções de um conjunto com n elementos em um conjunto com n elementos e SE considerarmos a definição de números binomiais em termos de

[obm-l] Re: [obm-l] RE: [obm-l] dúvida fatorial

2006-04-03 Por tôpico Ronaldo Luiz Alonso
Eu havia imaginado vagamente (a tempos atrás) tudo o que o professor Paulo colocou nesta mensagem (Show de Bola). Só que não tinha exemplos concretos nem clareza de idéias e também nem citações suficientes para explicitá-las como as que foram por ele colocadas. A moral disso tudo é que devem

[obm-l] Re: [obm-l] Re:[obm-l] Dúvida!

2006-02-04 Por tôpico Marcelo Salhab Brogliato
Olá, primeiramente vamos analisar o seguinte problema: x+y+z = k, x >= 0, y >= 0, z >= 0   Imaginemos que vc tem k palitos de sorvete e 2 pedras.. de quantos modos vc pode organiza-los? (k+2)! / (k! 2!), certo? que é igual a C(k+2, 2) .. combinação de k+2, tomados 2 a 2.   Agora, considere q

Re: [obm-l] RE: [obm-l] Re: [obm-l] Dúvida Raiz

2005-12-31 Por tôpico ricardo.bioni
O valor de x^(1/n) onde n é um número par é apenas o número positivo y tal que y^n = xSe for procurado o número negativo y tal que y^n = x então o valor procurado é y = -x^(1/n)

[obm-l] RE: [obm-l] Re: [obm-l] Dúvida Raiz

2005-12-31 Por tôpico Guilherme Neves
ah, completando minha resposta.. no campo dos complexos, utilize as formulas de De Moivre que você obterá as raízes com os dois sinais. = Instruções para entrar na lista, sair da lista e usar a lista em http://www.mat.puc-rio

[obm-l] Re: [obm-l] Re:[obm-l] Dúvida

2004-06-25 Por tôpico claudio.buffara
  > Meu caro Cláudio, >   > fiquei me perguntando sobre a seguinte afirmação: >   > "Mas A pode ser particionado em pares nao ordenados da forma: > {x,x^(-1)}" >   > O que garante que cada x pertencente a A tem seu inverso em A? >   A eh o conjunto dos elementos de G que sao diferentes dos respect

[obm-l] Re:[obm-l] RE: [obm-l] Dúvida

2004-06-24 Por tôpico claudio.buffara
  De: [EMAIL PROTECTED] Para: [EMAIL PROTECTED] Cópia: Data: Thu, 24 Jun 2004 14:38:52 -0300 Assunto: [obm-l] RE: [obm-l] Dúvida     > Você também está usando o fato do grupo ser abeliano, não? > > "Caso 2: pelo menos dois dos x_i sao distintos. > Nesse caso, a classe vai

[obm-l] RE: [obm-l] Re:[obm-l] RE: [obm-l] Dúvida

2004-06-24 Por tôpico Paulo Santa Rita
356,240604 From: "claudio.buffara" <[EMAIL PROTECTED]> Reply-To: [EMAIL PROTECTED] To: "obm-l" <[EMAIL PROTECTED]> Subject: [obm-l] Re:[obm-l] RE: [obm-l] Dúvida Date: Thu, 24 Jun 2004 12:43:59 -0300 Oi, Paulo: Acho que esta sua demonstracao do teorema de Cauchy soh

[obm-l] Re:[obm-l] RE: [obm-l] Dúvida

2004-06-24 Por tôpico claudio.buffara
Oi, Paulo:   Acho que esta sua demonstracao do teorema de Cauchy soh eh valida se G for abeliano, pois no fim, quando voce fala na projecao canonica p: G -> G/H, voce estah implicitamente supondo que G/H eh um grupo e, portanto, que H eh um subgrupo normal de G. Mas isso soh eh verdade para todo H

[obm-l] RE: [obm-l] Re:[obm-l] RE: [obm-l] Re:[obm-l] dúvida chara!

2004-05-23 Por tôpico Rogério Moraes de Carvalho
mesmo no conjunto dos números inteiros. Atenciosamente, Rogério Moraes de Carvalho -Original Message- From: [EMAIL PROTECTED] [mailto:[EMAIL PROTECTED] On Behalf Of Osvaldo Sent: domingo, 23 de maio de 2004 17:54 To: obm-l Subject: [obm-l] Re:[obm-l] RE: [obm-l] Re:[obm-l] dúvida chara! Descul

[obm-l] Re:[obm-l] RE: [obm-l] Re:[obm-l] dúvida chara!

2004-05-23 Por tôpico Osvaldo
Desculpe-me se fui parcial Dr., porém equivoquei-me ao ler o enunciado da questão. Eu apenas fiz os calculos para os números inteiros e não naturais, ou seja, inclui algumas possibilidades a mais. Obrigado pela observação! > Olá colegas da lista, > > Apesar da resolução apresentada

[obm-l] RE: [obm-l] Re:[obm-l] dúvida chara!

2004-05-23 Por tôpico Rogério Moraes de Carvalho
Olá colegas da lista, Apesar da resolução apresentada pelo Osvaldo ter seguido um possível raciocínio correto para resolver esta questão, a análise dele está incompleta porque omite alguns passos muito importantes, o que pode nos levar a encontrar soluções inválidas. Neste problema especif

[obm-l] Re: [obm-l] Re: [obm-l] dúvida PARA FAELCCMM

2004-05-08 Por tôpico Fernando Romualdo Braga
Se considerarmos apenas os cortes ADICIONAIS a partir do primeiro, a alternativa correta é a C- 6 e 25º, o total de cortes para se obter os 14 pedaços são 7.   - Original Message - From: TSD To: [EMAIL PROTECTED] Sent: Saturday, May 08, 2004 8:43 PM Subject: [obm-l] Re: [obm-l] dúvi

[obm-l] Re:[obm-l] Re: [obm-l] Re:[obm-l] dúvida

2004-04-25 Por tôpico rickufrj
-- Início da mensagem original --- De: [EMAIL PROTECTED] Para: [EMAIL PROTECTED] Cc: Data: Sun, 25 Apr 2004 10:10:46 -0300 Assunto: [obm-l] Re: [obm-l] Re:[obm-l] dúvida > No caso, não entendi o porque do i*(raiz de 1998), visto que ao elevarmos

[obm-l] Re: [obm-l] Re:[obm-l] dúvida

2004-04-25 Por tôpico Fellipe Rossi
No caso, não entendi o porque do i*(raiz de 1998), visto que ao elevarmos ao quadrado, i^2= -1 e a expressão seria -1998 Creio que apenas (raiz de 1998) seja mais correto. Porém a questão não deve ser apenas isto, a e b devem pertencer a algum conjunto específico como os Inteiros... Abraços, Ro

[obm-l] Re: [obm-l] Re: [obm-l] Re: [obm-l] Dúvida persistente!!!

2004-04-13 Por tôpico Rafael
um problema bonito. Cláudio, Parabéns por ambas as soluções! Abraços, Rafael de A. Sampaio - Original Message - From: "Qwert Smith" <[EMAIL PROTECTED]> To: <[EMAIL PROTECTED]> Sent: Tuesday, April 13, 2004 5:41 PM Subject: RE: [obm-l] Re: [obm-l] Re: [obm

RE: [obm-l] Re: [obm-l] Re: [obm-l] Dúvida persistente!!!

2004-04-13 Por tôpico Qwert Smith
Reply-To: [EMAIL PROTECTED] To: "OBM-L" <[EMAIL PROTECTED]> Subject: [obm-l] Re: [obm-l] Re: [obm-l] Dúvida persistente!!! Date: Tue, 13 Apr 2004 03:20:58 -0300 Eu desisto... Tentei encontrar uma solução simples, como pedia o Eduardo, mas a melhor forma que vejo agora é calcular, por

  1   2   >